Download as pptx, pdf, or txt
Download as pptx, pdf, or txt
You are on page 1of 144

CR Basics

p t a
Gu
e e p
a n d
S
CR Basics Part 1
p t a
Gu
e e p
a n d
S
a
Please note:

u p t
G
On each slide, check

e
the bottom

e p
d
notes

Sa n
for explanations,
wherever applicable.
a
Argument (core / flow)

u p t
PC
p G
d e e
Sa n
The argument is the entire passage
(also called stimulus)
a
Argument Types
• Pure facts, no conclusion

u p t
G
– usually inference questions
• Seemingly contradictory facts, no conclusion

e p
– usually paradox questions

e
• A few facts, one conclusion (standard argument – about 80% of the questions)

d
• Facts  IC (intermediate conclusion) – MC (main conclusion), usually boldface questions

n
• Two opposing conclusions / viewpoints (usually the author and some other side) with

a
facts backing both the claims, usually boldface questions

S
• Two paragraphs – two different arguments (one responding to the other), various types of
questions
a
Pure facts, no conclusion

u p t
G
Students from outside the province of Markland, who in any

p
given academic year pay twice as much tuition fee each as do

e
students from Markland, had traditionally accounted for at

d e
least two-thirds of the enrollment at Central Markland

n
College. Over the past 10 years academic standards at the

Sa
college have risen, and the proportion of students who are not
Marklanders has dropped to around 40 percent.
a
Seemingly contradictory facts, no conclusion

u p t
G
Liberty Gallery was a popular art gallery in Newport City for

p
many years. Due to budget constraints necessitated by a spike

e
in city taxes, the owners of the gallery were forced to move

d e
their company to the small suburb of Ebbertsville, a town with

n
a population twenty times smaller than Newport's. In spite of

Sa
the fact that the pedestrian traffic in Ebbertsville is
significantly lighter, Liberty Gallery has never served more
regular visitors and patrons.
a
A few facts, one conclusion

u p t
G
When people show signs of having a heart attack an electrocardiograph

p
(ECG) is often used to diagnose their condition. In a study, a computer

e
program for ECG diagnosis of heart attacks was pitted against a very

e
experienced, highly skilled cardiologist. The program correctly

n d
diagnosed a significantly higher proportion of the cases that were later

a
confirmed to be heart attacks than did the cardiologist. Interpreting

S
ECG data, therefore, should be left to computer programs alone.
a
Facts  IC  MC

u p t
G
Local authorities are considering an amendment to the litter law that

p
would raise the fine for littering in the community picnic area to

e
$1,000. Since the inception of the litter law, incremental increases in

e
the littering fine have proven to be consistently effective at further

n d
reducing the amount of litter in the community picnic area. However,

a
raising the fine to $1,000 would actually have the unintended effect of

S
increasing the amount of litter in the picnic area. Picnic area users
would perceive this fine to be unreasonable and unenforceable, and
would disregard the litter law altogether.
Two opposing conclusions / viewpoints (usually the author

t a
and some other side) with facts backing both the claims

Gu p
Historian: Newton developed mathematical concepts and techniques that are
fundamental to modern calculus. Leibniz developed closely analogous concepts and

p
techniques. It has traditionally been thought that these discoveries were independent.

e
Researchers have, however, recently discovered notes of Leibniz’ that discuss one of

d e
Newton’s books on mathematics. Several scholars have argued that since the book

n
includes a presentation of Newton’s calculus concepts and techniques, and since the notes

a
were written before Leibniz’ own development of calculus concepts and techniques, it is

S
virtually certain that the traditional view is false. A more cautious conclusion than this is
called for, however. Leibniz’ notes are limited to early sections of Newton’s book, sections
that precede the ones in which Newton’s calculus concepts and techniques are presented.
Two paragraphs – two different arguments (one responding to the other)

p t a
Jane: Professor Harper’s ideas for modifying the design of guitars are of no value because there is no general

u
agreement among musicians as to what a guitar should sound like and, consequently, no widely accepted

G
basis for evaluating the merits of a guitar’s sound.
Mark: What’s more, Harper’s ideas have had enough time to be adopted if they really resulted in superior

p
sound. It took only ten years for the Torres design for guitars to be almost universally adopted because of the

e
improvement it makes in tonal quality.

e
Which one of the following most accurately describes the relationship between Jane’s argument and Mark’s

d
argument?

a n
A. Mark’s argument shows how a weakness in Jane’s argument can be overcome.

S
B. Mark’s argument has a premise in common with Jane’s argument.
C. Mark and Jane use similar techniques to argue for different conclusions.
D. Mark’s argument restates Jane’s argument in other terms.
E. Mark’s argument and Jane’s argument are based on conflicting suppositions.
How to find the conclusion

p t a
u
Future | Feeling | Uncertainty | Debatability | Challengeability?

G
 Prediction … (may | might | can | could | will | would | likely etc.) – FUTURE

p
 Suggestion … (should | propose | recommend | make a case for | ‘would need to

e e
do’ | would be wise to follow etc. | must | ought to etc.) – FEELING / FUTURE

d
 Position | stand | stance | viewpoint | belief – FEELING / DEBATABILITY

a n
 Criticism or praise | Agreement or disagreement – FEELING

S
 Judgment reached | Hypothesis | Claim – DEBATABILITY / UNCERTAINTY

Conclusions can be challenged / supported.


t a
The Therefore Test

u p
This test instantly tells you which statement is the first in the order of logic (and hence is the

G
premise or the intermediate conclusion) and which statement is the last in the order of logic

p
(and hence is the final conclusion).

e
The test is: If we can logically write Because X, therefore Y

d e
Then X is the premise / intermediate conclusion and Y is the final conclusion.

n
Here the most important word is logically. Means the order must make sense.

Sa
Consider the example below:
A new lemonade stand has just opened for business in the town square. The stand
will surely fail. A popular juice store already sells lemonade in the town square, so the
new lemonade stand will not be able to attract customers.
a
 BACKGROUND INFORMATION: A new lemonade stand has just opened for
business in the town square.

u p t
G
 CONCLUSION (final opinion): The stand will surely fail.

e p
 SUPPORTING PREMISE (fact): A popular juice store already sells lemonade

d e
in the town square

a n
 INTERMEDIATE CONCLUSION (opinion): so, the new lemonade stand will

S
not be able to attract customers.
t a
Conclusion Cues: So, Thus, Therefore, Thereby, Consequently, Clearly, As a

u p
result, For this reason, This demonstrates that, They conclude that, Hence,

G
Accordingly, It must be that, It shows that, It follows that, It is likely that

e e p
d
Premise Cues: Since, The reason is, Because, For (when it means ‘because’), For

a n
example, For the reason that, In that, Given that, As indicated by, Due to, Owing

S
to, This can be seen from, We know this by
a
A special cue: Some people believe: One of the most frequently used constructions is to raise a viewpoint at the

p t
beginning of the stimulus and then disagree with it immediately thereafter. This efficiently raises two opposing

u
views in a very short paragraph. These stimuli are recognizable because they often begin with the phrase, “Some

G
people claim...” or one of the many variations on this theme, including but not limited to the following:

e p
“Some people propose...” “Many people believe...”

e
“Some people argue that...” “Some critics claim...”

n d
“Some critics maintain...” “Some scientists believe...

Sa
All these imply that the author MAY (not compulsory) challenge whatever is said by others.
a
Facts  IC  MC

u p t
G
Local authorities are considering an amendment to the litter law that

p
would raise the fine for littering in the community picnic area to

e
$1,000. Since the inception of the litter law, incremental increases in

e
the littering fine have proven to be consistently effective at further

n d
reducing the amount of litter in the community picnic area. However,

a
raising the fine to $1,000 would actually have the unintended effect of

S
increasing the amount of litter in the picnic area. Picnic area users
would perceive this fine to be unreasonable and unenforceable, and
would disregard the litter law altogether.
a
Facts  IC  MC

u p t
Local authorities are considering an amendment to the litter law that

G
would raise the fine for littering in the community picnic area to

p
$1,000. Since the inception of the litter law, incremental increases in

e
the littering fine have proven to be consistently effective at further

e
reducing the amount of litter in the community picnic area.

n d
MC: However, raising the fine to $1,000 would actually have the

a
unintended effect of increasing the amount of litter in the picnic area.

S
IC: Picnic area users would perceive this fine to be unreasonable and
unenforceable, and would disregard the litter law altogether.
a
Facts  IC  MC

u p t
G
The city government should invest surplus funds in improving the

p
city's transportation network. Most of the network was put in place at a

e
time when the city was much smaller in both area and population. The

e
subway system is outdated and understaffed. The buses rarely run on

n d
schedule and their routes are inconvenient. If the city does not make

a
changes soon to the network, it will see many of its prized industries

S
relocate to more convenient cities and, as a result, the city's financial
health will be jeopardized.
a
Facts  IC  MC

u p t
MC: The city government should invest surplus funds in improving the

G
city's transportation network.

p
Most of the network was put in place at a time when the city was much

e
smaller in both area and population. The subway system is outdated

d e
and understaffed. The buses rarely run on schedule and their routes

n
are inconvenient.

a
IC: If the city does not make changes soon to the network, it will see

S
many of its prized industries relocate to more convenient cities and, as
a result, the city's financial health will be jeopardized.
Boldface Questionsa
u p t
p G
d e e
Sa n
•Step 1: Identify the main conclusion of the author. No matter what, this is your first step.

a
•Step 2: Label each boldface statement as Premise or Conclusion

t
•Step 3: You must remember the other terms used for Premise and Conclusion

p
 Premise: (two types – challengeable and non-challengeable)

u
• NON-CHALLENGEABLE PREMISES: Fact, Truth, Data, Evidence, Information, Situation, Circumstance, Research,

G
Proof, Observation, Example, Finding, Phenomenon, Illustration, Pattern (in the past or present)
• CHALLENGEABLE PREMISES: Reason, Justification, Reasoning, Explanation, Line of Reasoning, Support,

p
Statement of support, Supposition, Consideration, Policy, Practice, Generalization (about past or present), Judgment in

e
support, Acknowledgement

e
 Conclusion: Judgment reached, Opinion, Suggestion, View, Idea, Belief, Proposal, Warning, Forecast, Claim, Stand,

d
Prediction, Hypothesis, Position (to posit), Stance, Point, Main Point, Generalization (about future), Contention,

n
Consequence, Pattern (guessed / predicted for the future)

a
•Never forget:

S
 The role of a premise is to support a conclusion
 We can never challenge or support a fact
• Step 4: Check the presence of a contradiction word between the two Boldface statements. The common contradiction words
are:
• But, However, Nonetheless, Nevertheless, Notwithstanding, Even so, Despite, Rather, Yet, On the other hand, Admittedly,
In contrast, By contrast, Contrary to, Although, Even though, Still, Whereas, In spite of, After all, Alternatively, Apart from,
Conversely, Regardless, Then again, Unfortunately, Ironically, while etc.
•STEP 5: Go to the options and eliminate
a
Please note:

u p t
G
On each slide, check

e
the bottom

e p
d
notes

Sa n
for explanations,
wherever applicable.
Historian: Newton developed mathematical concepts and techniques that are fundamental to modern

a
calculus. Leibniz developed closely analogous concepts and techniques. It has traditionally been thought

t
that these discoveries were independent. Researchers have, however, recently discovered notes of

p
Leibniz’ that discuss one of Newton’s books on mathematics. Several scholars have argued that since the

u
book includes a presentation of Newton’s calculus concepts and techniques, and since the notes were
written before Leibniz’ own development of calculus concepts and techniques, it is virtually certain

G
that the traditional view is false. A more cautious conclusion than this is called for, however. Leibniz’
notes are limited to early sections of Newton’s book, sections that precede the ones in which

p
Newton’s calculus concepts and techniques are presented.

e
In the historian’s reasoning, the two boldfaced portions play which of the following roles?

e
A. The first provides evidence in support of the overall position that the historian defends; the second
is evidence that has been used to support an opposing position.

d
B. The first provides evidence in support of the overall position that the historian defends; the second

n
is that position.

a
C. The first provides evidence in support of an intermediate conclusion that is drawn to provide
support for the overall position that the historian defends; the second provides evidence against that

S
intermediate conclusion.
D. The first is evidence that has been used to support a conclusion that the historian criticizes; the
second is evidence offered in support of the historian’s own position.
E. The first is evidence that has been used to support a conclusion that the historian criticizes; the
second is further information that substantiates that evidence.
Historian: Newton developed mathematical concepts and techniques that are fundamental to

a
modern calculus. Leibniz developed closely analogous concepts and techniques. It has traditionally

t
been thought that these discoveries were independent. Researchers have, however, recently

p
discovered notes of Leibniz’ that discuss one of Newton’s books on mathematics. Several scholars

u
have argued that P1: since the book includes a presentation of Newton’s calculus concepts and
techniques, and P2: since the notes were written before Leibniz’ own development of calculus

G
concepts and techniques, C1: it is virtually certain that the traditional view is false. C2: A more
cautious conclusion than this is called for, however. P3: Leibniz’ notes are limited to early sections

p
of Newton’s book, sections that precede the ones in which Newton’s calculus concepts and

e
techniques are presented.

e
In the historian’s reasoning, the two boldfaced portions play which of the following roles?
A. The first provides evidence in support of the overall position that the historian defends; the

d
second is evidence that has been used to support an opposing position.

n
B. The first provides evidence in support of the overall position that the historian defends; the

a
second is that position.
C. The first provides evidence in support of an intermediate conclusion that is drawn to provide

S
support for the overall position that the historian defends; the second provides evidence
against that intermediate conclusion.
D. The first is evidence that has been used to support a conclusion that the historian criticizes;
the second is evidence offered in support of the historian’s own position.
E. The first is evidence that has been used to support a conclusion that the historian criticizes;
the second is further information that substantiates that evidence.
Two opposing conclusions / viewpoints (usually the author

t a
and some other side) with facts backing both the claims

Gu p
e e p
a n d
S
Economist: Economists have long argued over the best way to measure the net wealth of a nation's

a
economy. On one side are those that favor the use of Gross Domestic Product, or GDP, the market

t
value of all goods and services produced within the borders of a country within one year. On the
other side are supporters of the use of the GNP, a similar measure that, unlike GDP, takes into account

p
foreign assets that are owned by residents of a country. The debate over the accuracy of the GDP versus

u
the GNP as a measure of net wealth continues, but in reality, the strong population bias associated with
both statistics likely renders both inaccurate. In truth, per-capita GDP offers the clearest picture, as it

G
divides the total wealth by the number of individuals, which tells us the mean income and average
wealth and lifestyle of a nation's people. The difference between GDP / GNP and per-capita GDP is

p
stark. According to the former measures, China has recently overtaken Japan as the second wealthiest

e
country. If we look at per-capita GDP, however, Japan, China, and even the United States do not

e
make the top ten, while small wealthy nations like Denmark, Switzerland, and Luxembourg
routinely rank higher.

d
In the economist's argument, the two portions in boldface play which of the following roles?

n
A. The first introduces one side of a dichotomy; the second offers a concrete example of the dichotomy.

a
B. The first expresses an opinion about one perspective on a dichotomy; the second describes premise
undermining the dichotomy itself.

S
C. The first describes a general topic; the second offers an example of an alternative approach to that
topic.
D. The first is a common argument about a subject; the second is the author's conclusion.
E. The first identifies one side of a dichotomy; the second offers an example showing the dichotomy to
be false.
Economist: Economists have long argued over the best way to measure the net wealth of a nation's

a
economy. P1: On one side are those that favor the use of Gross Domestic Product, or GDP, the

t
market value of all goods and services produced within the borders of a country within one
year. On the other side are supporters of the use of the GNP, a similar measure that, unlike GDP, takes

p
into account foreign assets that are owned by residents of a country. The debate over the accuracy of the

u
GDP versus the GNP as a measure of net wealth continues, but in reality, the strong population bias
associated with both statistics likely renders both inaccurate. In truth, per-capita GDP offers the

G
clearest picture, as it divides the total wealth by the number of individuals, which tells us the mean
income and average wealth and lifestyle of a nation's people. The difference between GDP / GNP and

p
per-capita GDP is stark. According to the former measures, China has recently overtaken Japan as the

e
second wealthiest country. P2: If we look at per-capita GDP, however, Japan, China, and even the

e
United States do not make the top ten, while small wealthy nations like Denmark, Switzerland,
and Luxembourg routinely rank higher.

d
In the economist's argument, the two portions in boldface play which of the following roles?

n
A. The first introduces one side of a dichotomy; the second offers a concrete example of the dichotomy.

a
B. The first expresses an opinion about one perspective on a dichotomy; the second describes premise
undermining the dichotomy itself.

S
C. The first describes a general topic; the second offers an example of an alternative approach to that
topic.
D. The first is a common argument about a subject; the second is the author's conclusion.
E. The first identifies one side of a dichotomy; the second offers an example showing the
dichotomy to be false.
Economist: Scholars in economics and public policy have long argued over the best way to measure

a
emotions such as love, anger, and jealousy in quantifiable terms that can both enhance businesses'

t
capacity to fulfill customer needs and allow the government to implement policies that best address
issues of national concern. On one side are those who believe scholars can conduct research on

p
emotions to such an extent that it is possible to discover the financial value an individual places

u
on such principles as health, the safety of one's children, and the worth of higher
education. Sophisticated preference studies assessing a population's decisions on a very atomic level are

G
used to conduct these investigations. On the other side are those that believe such studies, however
perfected, are best only vague estimates of how emotions factor into decisions. Those who support this

p
belief cite numerous examples of situations (an expected trend or pop culture success) in which a

e
population's actual behavior disrupted scholars' meticulously constructed mathematical

e
models. It remains to be seen whether mathematical models regarding decision sciences will ever reach
a higher degree of accuracy.

d
In the economist's argument, the two highlighted portions play which of the following roles?

n
A. The first introduces one side of a dichotomy; the second offers a concrete example of that dichotomy.

a
B. The first discusses a common viewpoint about a dichotomy; the second describes premise

S
undermining the dichotomy itself.
C. The first describes a general topic; the second offers an example of an alternative approach to that
topic.
D. The first is a common argument about a subject; the second is the author's conclusion.
E. The first identifies one side of a dichotomy; the second indicates how one side supports its argument.
Economist: Scholars in economics and public policy have long argued over the best way to measure

a
emotions such as love, anger, and jealousy in quantifiable terms that can both enhance businesses'

t
capacity to fulfill customer needs and allow the government to implement policies that best address
issues of national concern. On one side are those who believe scholars can conduct research on

p
emotions to such an extent that it is possible to discover the financial value an individual places

u
on such principles as health, the safety of one's children, and the worth of higher
education. Sophisticated preference studies assessing a population's decisions on a very atomic level are

G
used to conduct these investigations. On the other side are those that believe such studies, however
perfected, are best only vague estimates of how emotions factor into decisions. Those who support

p
this belief cite numerous examples of situations (an expected trend or pop culture success) in

e
which a population's actual behavior disrupted scholars' meticulously constructed mathematical

e
models. It remains to be seen whether mathematical models regarding decision sciences will ever reach
a higher degree of accuracy.

d
In the economist's argument, the two highlighted portions play which of the following roles?

n
A. The first introduces one side of a dichotomy; the second offers a concrete example of that dichotomy.

a
B. The first discusses a common viewpoint about a dichotomy; the second describes premise

S
undermining the dichotomy itself.
C. The first describes a general topic; the second offers an example of an alternative approach to that
topic.
D. The first is a common argument about a subject; the second is the author's conclusion.
E. The first identifies one side of a dichotomy; the second indicates how one side supports its argument.
p t a
Gu
p
Questions for Class discussion

d e e
Sa n
1. Historian: In the Drindian Empire, censuses were conducted annually to determine the

a
population of each village. Village census records for the last half of the 1600’s are

t
remarkably complete. This very completeness makes one point stand out; in five different

p
years, villages overwhelmingly reported significant population declines. Tellingly, each of
those five years immediately followed an increase in a certain Drindian tax. This tax, which

u
was assessed on villages, was computed by the central government using the annual census

G
figures. Obviously, whenever the tax went up, villages had an especially powerful economic
incentive to minimize the number of people they recorded; and concealing the size of a

p
village’s population from government census takers would have been easy. Therefore, it is

e
reasonable to think that the reported declines did not happen. In the historian’s argument,
the two portions in boldface play which of the following roles?

e
A. The first supplies a context for the historian’s argument; the second acknowledges a

d
consideration that has been used to argue against the position the historian seeks to establish.

n
B. The first presents evidence to support the position that the historian seeks to establish; the

a
second acknowledges a consideration that has been used to argue against that position.
C. The first provides a context for certain evidence that supports the position that the historian

S
seeks to establish; the second is that position.
D. The first is a position for which the historian argues; the second is an assumption that serves
as the basis of that argument.
E. The first is an assumption that the historian explicitly makes in arguing for a certain position;
the second acknowledges a consideration that calls that assumption into question.
1. Historian: In the Drindian Empire, censuses were conducted annually to determine the

a
population of each village. Village census records for the last half of the 1600’s are

t
remarkably complete. This very completeness makes one point stand out; in five different

p
years, villages overwhelmingly reported significant population declines. Tellingly, each of
those five years immediately followed an increase in a certain Drindian tax. This tax, which

u
was assessed on villages, was computed by the central government using the annual census

G
figures. Obviously, whenever the tax went up, villages had an especially powerful economic
incentive to minimize the number of people they recorded; and concealing the size of a

p
village’s population from government census takers would have been easy. Therefore, it is

e
reasonable to think that the reported declines did not happen. In the historian’s argument,
the two portions in boldface play which of the following roles?

e
A. The first supplies a context for the historian’s argument; the second acknowledges a

d
consideration that has been used to argue against the position the historian seeks to establish.

n
B. The first presents evidence to support the position that the historian seeks to establish; the

a
second acknowledges a consideration that has been used to argue against that position.
C. The first provides a context for certain evidence that supports the position that the

S
historian seeks to establish; the second is that position.
D. The first is a position for which the historian argues; the second is an assumption that serves
as the basis of that argument.
E. The first is an assumption that the historian explicitly makes in arguing for a certain position;
the second acknowledges a consideration that calls that assumption into question.
2. Astronomer: Observations of the Shoemaker-Levi comet on its collision course with Jupiter

a
showed that the comet broke into fragments before entering Jupiter’s atmosphere in 1994,

t
but they did not show how big those fragments were. In hopes of gaining some indication of

p
the fragments’ size, astronomers studied spectrographic analyses of Jupiter’s outer
atmosphere. These analyses revealed unprecedented traces of sulfur after the fragments’

u
entry. Surprisingly, the fragments themselves certainly contained no sulfur. Many

G
astronomers consequently believe that the cloud layer below Jupiter’s outer atmosphere does
contain sulfur. Since sulfur would have seeped into the outer atmosphere if comet fragments

p
had penetrated this cloud layer, it is likely that some of the fragments were at least large

e
enough to have passed through Jupiter’s outer atmosphere without being burned up.

e
In the astronomer’s argument, the two portions in boldface play which of the following roles?

d
A. The first presents a circumstance for which the astronomer offers an explanation; the second

n
part is that explanation.
B. The first acknowledges a consideration that weighs against the conclusion of the argument;

a
the second is that conclusion.

S
C. The first acknowledges a consideration that weights against the conclusion of the argument;
the second provides evidence in support of that conclusion.
D. The first provides evidence in support of the conclusion of the argument; the second
acknowledges a consideration that weighs against that conclusion.
E. The first is a judgment advanced in support of the conclusion of the argument; the second is
that conclusion.
2. Astronomer: Observations of the Shoemaker-Levi comet on its collision course with Jupiter

a
showed that the comet broke into fragments before entering Jupiter’s atmosphere in 1994,

t
but they did not show how big those fragments were. In hopes of gaining some indication of

p
the fragments’ size, astronomers studied spectrographic analyses of Jupiter’s outer
atmosphere. These analyses revealed unprecedented traces of sulfur after the fragments’

u
entry. Surprisingly, the fragments themselves certainly contained no sulfur. Many

G
astronomers consequently believe that the cloud layer below Jupiter’s outer atmosphere does
contain sulfur. Since sulfur would have seeped into the outer atmosphere if comet fragments

p
had penetrated this cloud layer, it is likely that some of the fragments were at least large

e
enough to have passed through Jupiter’s outer atmosphere without being burned up.

e
In the astronomer’s argument, the two portions in boldface play which of the following roles?

d
A. The first presents a circumstance for which the astronomer offers an explanation; the second

n
part is that explanation.
B. The first acknowledges a consideration that weighs against the conclusion of the argument;

a
the second is that conclusion.

S
C. The first acknowledges a consideration that weights against the conclusion of the argument;
the second provides evidence in support of that conclusion.
D. The first provides evidence in support of the conclusion of the argument; the second
acknowledges a consideration that weighs against that conclusion.
E. The first is a judgment advanced in support of the conclusion of the argument; the
second is that conclusion.
3. City Official: At City Hospital, uninsured patients tend to have shorter stays and fewer

a
procedures performed than do insured patients, even though insured patients, on

t
average, have slightly less serious medical problems at the time of admission to the

p
hospital than uninsured patients have. Critics of the hospital have concluded that the

u
uninsured patients are not receiving proper medical care. However, this
conclusion is almost certainly false. Careful investigation has recently shown two

G
things: insured patients have much longer stays in the hospital than necessary, and

p
they tend to have more procedures performed than are medically necessary. In the
city official’s argument, the two boldface portions play which of the following roles?

e
A. The first states the conclusion of the city official’s argument; the second provides

e
support for that conclusion.

d
B. The first is used to support the conclusion of the city official’s argument; the second

n
states that conclusion.

a
C. The first was used to support the conclusion drawn by hospital critics; the second

S
states the position that the city official’s argument opposes.
D. The first was used to support the conclusion drawn by hospital critics; the second
provides support for the conclusion of the city official’s argument.
E. The first states the position that the city official’s argument opposes; the second states
the conclusion of the city official’s argument.
3. City Official: At City Hospital, uninsured patients tend to have shorter stays and fewer

a
procedures performed than do insured patients, even though insured patients, on

t
average, have slightly less serious medical problems at the time of admission to the

p
hospital than uninsured patients have. Critics of the hospital have concluded that the

u
uninsured patients are not receiving proper medical care. However, this
conclusion is almost certainly false. Careful investigation has recently shown two

G
things: insured patients have much longer stays in the hospital than necessary, and

p
they tend to have more procedures performed than are medically necessary. In the
city official’s argument, the two boldface portions play which of the following roles?

e
A. The first states the conclusion of the city official’s argument; the second provides

e
support for that conclusion.

d
B. The first is used to support the conclusion of the city official’s argument; the second

n
states that conclusion.

a
C. The first was used to support the conclusion drawn by hospital critics; the second

S
states the position that the city official’s argument opposes.
D. The first was used to support the conclusion drawn by hospital critics; the second
provides support for the conclusion of the city official’s argument.
E. The first states the position that the city official’s argument opposes; the second
states the conclusion of the city official’s argument.
4. Delta products, Inc., has recently switched at least partly from older

a
technologies using fossil fuels to new technologies powered by

t
electricity. The question has been raised whether it can be concluded that for

p
a given level of output, Delta’s operation now causes less fossil fuel to be

u
consumed than it did formerly. The answer, clearly, is yes, since the amount of

G
fossil fuel used to generate the electricity needed to power the new
technologies is less than the amount needed to power the older

p
technologies, provided that the level of output is held constant. In the

e
argument given, the two boldface portions play which of the following roles?

e
A. The first is the conclusion of the argument; the second provides support for

d
that conclusion.

n
B. The first provides support for the conclusion of the argument; the second is

a
the conclusion of the argument.

S
C. The first states the position that the argument opposes; the second states the
conclusion of the argument.
D. The first states the position that the argument challenges; the second states
the evidence in support of that challenge.
E. Each provides support for the conclusion of the argument.
4. Delta products, Inc., has recently switched at least partly from older

a
technologies using fossil fuels to new technologies powered by

t
electricity. The question has been raised whether it can be concluded that for

p
a given level of output, Delta’s operation now causes less fossil fuel to be

u
consumed than it did formerly. The answer, clearly, is yes, since the amount of

G
fossil fuel used to generate the electricity needed to power the new
technologies is less than the amount needed to power the older

p
technologies, provided that the level of output is held constant. In the

e
argument given, the two boldface portions play which of the following roles?

e
A. The first is the conclusion of the argument; the second provides support for

d
that conclusion.

n
B. The first provides support for the conclusion of the argument; the second is

a
the conclusion of the argument.

S
C. The first states the position that the argument opposes; the second states the
conclusion of the argument.
D. The first states the position that the argument challenges; the second states
the evidence in support of that challenge.
E. Each provides support for the conclusion of the argument.
5. Ecologist: The Scottish Highlands were once the site of extensive forests, but these forests

a
have mostly disappeared and been replaced by peat bogs. The common view is that the

t
Highlands’ deforestation was caused by human activity, especially agriculture. However,

p
agriculture began in the Highlands less than 2,000 years ago. Peat bogs, which consist
of compressed decayed vegetable matter, build up by only about one foot per 1,000 years

u
and, throughout the Highlands, remains of trees in peat bogs are almost all at depths

G
great than four feet. Since climate changes that occurred between 7,000 and 4,000 years
ago favored the development of peat bogs rather than the survival of forests, the

p
deforestation was more likely the result of natural processes than of human activity. In the

e
ecologist’s argument the two portions in boldface play which of the following roles?

e
A. The first is evidence that has been used in support of a position that the ecologist rejects;

d
the second is a finding that the ecologist uses to counter that evidence.
B. The first is evidence that, in light of the evidence provided in the second, serves as grounds

n
for the ecologist’s rejection of a certain position.

a
C. The first is a position that the ecologist rejects; the second is evidence that has been used

S
in support of that position.
D. The first is a position that the ecologist rejects; the second provides evidence in support of
that rejection.
E. The first is a position for which the ecologist argues; the second provides evidence to
support that position.
5. Ecologist: The Scottish Highlands were once the site of extensive forests, but these forests

a
have mostly disappeared and been replaced by peat bogs. The common view is that the

t
Highlands’ deforestation was caused by human activity, especially agriculture. However,

p
agriculture began in the Highlands less than 2,000 years ago. Peat bogs, which consist
of compressed decayed vegetable matter, build up by only about one foot per 1,000 years

u
and, throughout the Highlands, remains of trees in peat bogs are almost all at depths

G
great than four feet. Since climate changes that occurred between 7,000 and 4,000 years
ago favored the development of peat bogs rather than the survival of forests, the

p
deforestation was more likely the result of natural processes than of human activity. In the

e
ecologist’s argument the two portions in boldface play which of the following roles?

e
A. The first is evidence that has been used in support of a position that the ecologist rejects;

d
the second is a finding that the ecologist uses to counter that evidence.
B. The first is evidence that, in light of the evidence provided in the second, serves as

n
grounds for the ecologist’s rejection of a certain position.

a
C. The first is a position that the ecologist rejects; the second is evidence that has been used

S
in support of that position.
D. The first is a position that the ecologist rejects; the second provides evidence in support of
that rejection.
E. The first is a position for which the ecologist argues; the second provides evidence to
support that position.
6. Scientists typically do their most creative work before the age of forty. It is commonly

a
thought that this happens because aging by itself brings about a loss of creative

t
capacity. However, studies show that a disproportionately large number of the

p
scientists who produce highly creative work beyond the age of forty entered their
field at an older age than is usual. Since by the age of forty the large majority of scientists

u
have been working in their field for at least fifteen years, the studies’ finding strongly

G
suggests that the real reason why scientists over forty rarely produce highly creative work
is not that they have simply aged but rather that they generally have spent too long in a

p
given field. In the argument given, the two portions in boldface play which of the

e
following roles?

e
A. The first is the position that the argument as a whole opposes; the second is an objection

d
that has been raised against a position defended – wrong – there is no one defending any
position (conclusion) in the argument.

n
B. The first is a claim that has been advanced in support of a position that the argument

a
opposes; the second is a finding that has been used in support of that position.

S
C. The first is an explanation that the argument challenges; the second is a finding that has
been used in support of explanation presented in the first.
D. The first is an explanation that the argument challenges; the second is a finding on which
that challenge is based.
E. The first is an explanation that the argument defends; the second is a finding that has been
used to challenge that explanation.
6. Scientists typically do their most creative work before the age of forty. It is commonly

a
thought that this happens because aging by itself brings about a loss of creative

t
capacity. However, studies show that a disproportionately large number of the

p
scientists who produce highly creative work beyond the age of forty entered their
field at an older age than is usual. Since by the age of forty the large majority of scientists

u
have been working in their field for at least fifteen years, the studies’ finding strongly

G
suggests that the real reason why scientists over forty rarely produce highly creative work
is not that they have simply aged but rather that they generally have spent too long in a

p
given field. In the argument given, the two portions in boldface play which of the

e
following roles?

e
A. The first is the position that the argument as a whole opposes; the second is an objection

d
that has been raised against a position defended – wrong – there is no one defending any
position (conclusion) in the argument.

n
B. The first is a claim that has been advanced in support of a position that the argument

a
opposes; the second is a finding that has been used in support of that position.

S
C. The first is an explanation that the argument challenges; the second is a finding that has
been used in support of explanation presented in the first.
D. The first is an explanation that the argument challenges; the second is a finding on
which that challenge is based.
E. The first is an explanation that the argument defends; the second is a finding that has been
used to challenge that explanation.
7. Several of a certain bank’s top executives have recently been purchasing shares in their

a
own bank. This activity has occasioned some surprise, since it is widely believed that the

t
bank, carrying a large number of bad loans, is on the brink of collapse. Since the executives

p
are well placed to know their bank’s true condition, it might seem that their share purchases
show that the danger of collapse is exaggerated. The available information about the bank’s

u
condition is from reliable and informed sources, and corporate executives do sometimes buy

G
shares in their own company in a calculated attempt to calm worries about their company’s
condition. On balance, therefore, it is likely that the executives of the bank are following

p
this example. In the argument given, the two boldfaced portions play which of the following

e
roles?

e
A. The first describes the circumstance the explanation of which is the issue that the argument
addresses; the second states the main conclusion of the argument.

n d
B. The first describes the circumstance the explanation of which is the issue the argument
addresses; the second states a conclusion that is drawn in order to support the main

a
conclusion of the argument.

S
C. The first provides evidence to defend the position that the argument seeks to establish against
opposing positions; the second states the evidence that challenges the main conclusion of the
argument.
D. The first provides evidence to support the position that the argument seeks to establish; the
second states a conclusion that is drawn in order to support the argument’s main conclusion.
E. Each provides evidence to support the position that the argument seeks to establish.
7. Several of a certain bank’s top executives have recently been purchasing shares in their

a
own bank. This activity has occasioned some surprise, since it is widely believed that the

t
bank, carrying a large number of bad loans, is on the brink of collapse. Since the executives

p
are well placed to know their bank’s true condition, it might seem that their share purchases
show that the danger of collapse is exaggerated. The available information about the bank’s

u
condition is from reliable and informed sources, and corporate executives do sometimes buy

G
shares in their own company in a calculated attempt to calm worries about their company’s
condition. On balance, therefore, it is likely that the executives of the bank are following

p
this example. In the argument given, the two boldfaced portions play which of the following

e
roles?

e
A. The first describes the circumstance the explanation of which is the issue that the
argument addresses; the second states the main conclusion of the argument.

n d
B. The first describes the circumstance the explanation of which is the issue the argument
addresses; the second states a conclusion that is drawn in order to support the main

a
conclusion of the argument.

S
C. The first provides evidence to defend the position that the argument seeks to establish against
opposing positions; the second states the evidence that challenges the main conclusion of the
argument.
D. The first provides evidence to support the position that the argument seeks to establish; the
second states a conclusion that is drawn in order to support the argument’s main conclusion.
E. Each provides evidence to support the position that the argument seeks to establish.
8. As a large corporation in a small country, Hachnut wants its managers to have international

a
experience, so each year it sponsors management education abroad for its management

t
trainees. Hachnut has found, however, that the attrition rate of graduates from this program

p
is very high, with many of them leaving Hachnut to join competing firms soon after

u
completing the program. Hachnut does use performance during the program as a criterion in
deciding among candidates for management positions, but both this function and the goal

G
of providing international experience could be achieved in other ways. Therefore, if the
attrition problem cannot be successfully addressed, Hachnut should discontinue the

p
sponsorship program. In the argument given, the two boldfaced portions play which of the

e
following roles?

e
A. The first describes a practice that the argument seeks to justify; the second states a

d
judgment that is used in support of a justification for that practice.

n
B. The first describes a practice that the argument seeks to explain; the second presents part

a
of the argument’s explanation of that practice.

S
C. The first introduces a practice that the argument seeks to evaluate; the second provides
grounds for holding that the practice can never achieve its objective.
D. The first introduces a policy that the argument seeks to evaluate; the second provides
grounds for holding that the policy is not needed.
E. The first introduces a consideration supporting a policy that the argument seeks to
evaluate; the second provides evidence for concluding that the policy should be abandoned.
8. As a large corporation in a small country, Hachnut wants its managers to have international

a
experience, so each year it sponsors management education abroad for its management

t
trainees. Hachnut has found, however, that the attrition rate of graduates from this program

p
is very high, with many of them leaving Hachnut to join competing firms soon after

u
completing the program. Hachnut does use performance during the program as a criterion in
deciding among candidates for management positions, but both this function and the goal

G
of providing international experience could be achieved in other ways. Therefore, if the
attrition problem cannot be successfully addressed, Hachnut should discontinue the

p
sponsorship program. In the argument given, the two boldfaced portions play which of the

e
following roles?

e
A. The first describes a practice that the argument seeks to justify; the second states a

d
judgment that is used in support of a justification for that practice.

n
B. The first describes a practice that the argument seeks to explain; the second presents part

a
of the argument’s explanation of that practice.

S
C. The first introduces a practice that the argument seeks to evaluate; the second provides
grounds for holding that the practice can never achieve its objective.
D. The first introduces a policy that the argument seeks to evaluate; the second
provides grounds for holding that the policy is not needed.
E. The first introduces a consideration supporting a policy that the argument seeks to
evaluate; the second provides evidence for concluding that the policy should be abandoned.
a
Further study to CR Basics 1

u p t
p G
CR Topicwise PDF page 604

d e e
onwards … till the very end

Sa n
p t a
Gu
e e p
a n d
S
CR Basics Part 2
p t a
Gu
e e p
a n d
S
a
Cause and Effect Reasoning

p t
(present in about 10% of CR questions)

u
p G
d e e
Sa n
a
Please note:

u p t
G
On each slide, check

e
the bottom

e p
d
notes

Sa n
for explanations,
wherever applicable.
a
Cause and effect reasoning: If the conclusion an argument is: A causes / caused B

p t
The correct weakeners are:

Gu
 B caused A

p
 C caused B

e e
 C caused both A and B (e.g., hot weather may cause both beer sales and ice cream

n d
sales to increase)

Sa
a
Cause and effect reasoning: If the conclusion is: A causes B

p
The correct strengtheners (and the correct assumptions) are:

u t
G
 B did not cause A

e p
 C did not cause B

d e
 C did not cause both A and B

n
Sa
a
1. Studies indicate that older antelope are, on average, more cautious than younger

p t
antelope. This proves that getting older causes antelope to become more cautious.

u
Weaken?

G
2. Children with divorced parents exhibit 50% more behavioral problems than do

e p
children whose parents remain married. Therefore, divorce causes behavioral

d e
problems in children. Weaken?

a n
3. A recent study showed that individuals with abnormally low concentrations of vitamin

S
B6 in their bloodstream are three times as likely to suffer from disease Q. Therefore,
increasing the concentration of vitamin B6 within an individual's bloodstream can
protect that individual from disease. ASSUMPTION?
a
4. For the last five years, students at Hazelton High School have been required to wear

p t
strict uniforms while in school. During that same period, Hazelton's students scored

u
34% higher on standardized tests than in the preceding five-year period. Weaken?

p G
5. Manager X recently observed that Employee Y has been consistently arriving to work

e
an hour late for the last two weeks. Over the same time period, performance reports

d e
indicated that the quality of Employee Y's work product had decreased substantially.

a n
In an effort to reverse this decrease in performance, Manager X instituted harsh

S
penalties for arriving to work late. Weaken?
a
6. The Xanatic Eatery on Main Street has recently received the highest rating in the city's

p t
restaurant review guide. Several celebrities, among them the film star Bronco Cantrell,

u
are regular patrons of Xanatic. Therefore, the high rating received by Xanatic in the

G
restaurant review guide must be due to the presence of celebrities as regular patrons.

e p
Weaken?

n d e
Sa
In the last 20 years, there has been a significant increase in coffee

t a
consumption. During this same time period, there has been an increasing

u p
number of public coffee shops in urban areas. Therefore, the increase in

G
the number of public coffee shops must have caused the increase in

p
coffee consumption. Weaken?

d e e
Sa n
a
A study indicated that adults who listen to classical music regularly

u p t
are less likely to have anxiety disorders. Clearly, classical music

G
helps calm the nerves and lowers anxiety. Weaken?

e e p
a n d
S
Gotham City has seen a rise in crime over the past 5 years. The

p t a
chief of police has recently installed video surveillance cameras at

u
all major intersections in the neighborhoods with the highest crime

p G
rate. We can now expect to see a drop in the crime rate. Weaken?

d e e
Sa n
In the last decade there has been a significant decrease in coffee consumption. During this same time,

t a
there has been increasing publicity about the adverse long-term effects on health from the caffeine in

p
coffee. Therefore, the decrease in coffee consumption must have been caused by consumers’ awareness

u
of the harmful effects of caffeine.

G
Which of the following, if true, most seriously calls into question the explanation above?

p
A. On average, people consume 30% less coffee than they did 10 years ago.

e e
B. Heavy coffee drinkers may have mild withdrawal symptoms, such as headaches, for a day or so after,

d
significantly decreasing their coffee consumption.

n
C. Sales of specialty types of coffee have held steady, as sales of regular brands have declined.

Sa
D. The consumption of fruit juices and caffeine-free herbal teas has decreased over the past decade.
E. Coffee prices increased steadily in the past decade because of unusually severe frosts in coffee-
growing nations.
Journalist: In physics journals, the number of articles reporting the results of

a
experiments involving particle accelerators was lower last year than it had been in

t
previous years. Several of the particle accelerators at major research institutions

p
were out of service the year before last for repairs, so it is likely that the low number

u
of articles was due to the decline in availability of particle accelerators.

G
Which of the following, if true, most seriously undermines the journalist’s
argument?

p
A. Every article based on experiments with particle accelerators that was submitted

e
for publication last year actually was published.

e
B. The average time scientists must wait for access to a particle accelerator has

d
declined over the last several years.

n
C. The number of physics journals was the same last year as in previous years.

a
D. Particle accelerators can be used for more than one group of experiments in any

S
given year.
E. Recent changes in the editorial policies of several physics journals have
decreased the likelihood that articles concerning particle-accelerator research
will be accepted for publication.
A study of marital relationships in which one partner's sleeping and waking cycles

a
differ from those of the other partner reveals that such couples share fewer

t
activities with each other and have more violent arguments than do couples in a

p
relationship in which both partners follow the same sleeping and waking patterns.

u
Thus, mismatched sleeping and waking cycles can seriously jeopardize a marriage.

G
Which of the following, if true, most seriously weakens the argument above?
A. The sleeping and waking cycles of individuals tend to vary from season to season.

p
B. The individuals who have sleeping and waking cycles that differ significantly

e
from those of their spouses tend to argue little with colleagues at work.

e
C. People in unhappy marriages have been found to express hostility by adopting a

d
different sleeping and waking cycle from that of their spouses.

n
D. According to a recent study, most people's sleeping and waking cycles can be

a
controlled and modified easily.

S
Relationship Advisor: When people ask me about causes of divorce, I point to

t a
statistics. Surveys of couples have shown that those who live together before

p
getting married are more likely to get divorced. Clearly, living together before

u
marriage is one of the causes of divorce.
Which of the following, if true, is most damaging to the relationship

G
advisor’s conclusion?

p
A. In comparison with more conservative minded people, more liberal minded

e
people tend to be both more open to living together before marriage and

e
more open to getting divorced.

d
B. In some countries the divorce rate has steadily decreased over the past 30

n
years.

a
C. Couples who live together before getting married have an opportunity to

S
learn to work together as a team before they get married.
D. For preventing divorce, there is no substitute for having good communication
skills.
E. For many couples who live together before getting married, marriage does not
immediately lead to significant changes in their lifestyles.
Emotional stress is a well-known cause of certain serious health problems, including high

a
blood pressure and cardiac complications. Now, an additional concern can be added to the

t
list of maladies caused by emotional stress. A recent study surveyed both people who

p
have high levels of emotional stress and people who don’t, and found that people with

u
high emotional stress levels are significantly more obese and nervous than people with

G
lower levels of emotional stress.
Which of the following is an assumption on which the argument rests?

p
A. Obesity and nervousness can lead to serious health problems.

e
B. Emotionally stressed-out people are aware of the various health problems attributed

e
to emotional stress, including high blood pressure and cardiac complications.

d
C. Equivalent numbers of people with high and low levels of emotional stress were

n
surveyed for the study.

a
D. Obesity and nervousness do not make individuals less capable to deal with
emotionally stressful situations.

S
E. Emotionally stressed-out people who had encountered an emotionally stressful
situation immediately before responding to the survey were more obese and nervous
than the people form same group who had not encountered any emotionally stressful
situation for a few days.
In a study conducted in Pennsylvania, servers in various restaurants wrote “Thank

a
you” on randomly selected bills before presenting the bills to their customers. Tips

t
on these bills were an average of three percentage points higher than tips on bills

p
without the message. Therefore, if servers in Pennsylvania regularly wrote “Thank

u
you” on restaurant bills, their average income from tips would be significantly higher

G
than it otherwise would have been.
Which of the following is an assumption on which the argument relies?

p
A. The “Thank you” messages would not have the same impact on regular patrons of

e
a restaurant as they would on occasional patrons of the same restaurant

e
B. Regularly seeing “Thank you” written on their bills would not lead restaurant

d
patrons to revert to their earlier tipping habits

n
C. The written “Thank you” reminds restaurant patrons that tips constitute a

a
significant part of the income of many food servers

S
D. The rate at which people tip food servers in Pennsylvania does not vary with how
expensive a restaurant is
E. No patrons of the Pennsylvania restaurants in the study who were given a bill
with “Thank you” written on it left a larger tip than they otherwise would have.
It is true of both men and women that those who marry as young adults live longer

a
than those who never marry. This does not show that marriage causes people to live

t
longer, since, as compared with other people of the same age, young adults who are

p
about to get married have fewer of the unhealthy habits that can cause a person to

u
have a shorter life, most notably smoking and immoderate drinking of alcohol.

G
Which of the following, if true, most strengthens the argument above?
A. Marriage tends to cause people to engage less regularly in sports that involve risk

p
of bodily harm.

e
B. A married person who has an unhealthy habit is more likely to give up that habit

e
than a person with the same habit who is unmarried.

d
C. A person who smokes is much more likely than a nonsmoker to marry a person

n
who smokes at the time of marriage, and the same is true for people who drink

a
alcohol immoderately.

S
D. Among people who marry as young adults, most of those who give up an
unhealthy habit after marriage do not resume the habit later in life.
E. Among people who as young adults neither drink alcohol immoderately nor
smoke, those who never marry live as long as those who marry.
Medical researchers discovered that people with an atypical form of

t a
diabetes also have a disproportionately high rate of virus V infection.

p
They concluded that virus V triggers the mutation of classic diabetes into

u
its atypical form.

G
The researchers’ conclusion depends on which of the following
assumptions?

p
A. The longer the length of time an individual is infected with virus V, the

e
greater the risk that individual will develop atypical diabetes.

e
B. Atypical diabetes is a symptom of virus V.

n d
C. Individuals infected with virus V cannot develop atypical diabetes.

a
D. Atypical diabetes does not predispose people to infection from virus V.

S
E. Specialized treatment for virus V infection will be more effective than
the standard course of virus V treatment for people with atypical
diabetes.
Which of the following most logically completes the argument given?

a
Asthma, a chronic breathing disorder, is significantly more common today among adult

t
competitive swimmers than it is among competitive athletes who specialize in other

p
sports. Although chlorine is now known to be a lung irritant and swimming pool water is

u
generally chlorinated, it would be rash to assume that frequent exposure to chlorine is the

G
explanation of the high incidence of asthma among these swimmers, since __________.
A. young people who have asthma are no more likely to become competitive athletes

p
than are young people who do not have asthma

e
B. competitive athletes who specialize in sports other than swimming are rarely exposed

e
to chlorine

d
C. competitive athletes as a group have a significantly lower incidence of asthma than do

n
people who do not participate in competitive athletics

a
D. until a few years ago, physicians routinely recommended competitive swimming to
children with asthma, in the belief that this form of exercise could alleviate asthma

S
symptoms
E. many people have asthma without knowing they have it and thus are not diagnosed
with the condition until they begin engaging in very strenuous activities, such as
competitive athletics
a
NOTE … The following questions may have

p t
only one or two or three or four or five

u
G
options ... it is intentional.

e e p
a n d
S
When people engage in activities that help others, their brain releases

t a
endorphins, the brain's natural opiates, which induce in people a feeling of

u p
well-being. It has been suggested that regular release of endorphins

G
increases people's longevity. And a statistic on adults who regularly engage

p
in volunteer work helping others shows that they live longer, on average,

e e
than adults who do not volunteer. It is evident, then, that an act of

d
volunteering can boost longevity.

a n
Which of the following will weaken the argument?

S
A. In the communities studied, all volunteers were women, and it is
known, medically and statistically, that women live longer than the
overall average population.
Springfield Central School District's new superintendent recently instituted

t a
a controversial new plan to reduce discipline problems and improve school

p
spirit at the district's secondary schools. After observing that East Asian

u
countries require students to wear uniforms and that these schools also

G
suffer from dramatically fewer discipline problems than do American

p
schools, the superintendent instituted a mandatory uniform policy in all

e
district secondary schools. Since all students are now required to wear a

d e
formal uniform to class, discipline problems should decrease and the

n
students' morale should also improve. Which of the following, if true,

a
most seriously calls into question the conclusion of the argument

S
above?
A. East Asian schools use a different standard in their accounting of
discipline problems than do American schools.
The number of people diagnosed with tuberculosis in a certain city

t a
decreased significantly from one year to the next. Epidemiologists

p
attributed this decline to the closing of a coal plant on the outskirts

u
of the city; coal fumes have long been believed to cause acute lung

G
inflammation that can lead to the contraction of tuberculosis.

e p
Which of the following, if true, would most seriously weaken

e
the epidemiologists' explanation for the decrease in

d
tuberculosis diagnoses?

a n
A. Due to a new medical technique introduced into the city this year,

S
many people who would have been diagnosed with tuberculosis
are now correctly diagnosed with emphysema.
In modern deep-diving marine mammals, such as whales, the

t a
outer shell of the bones is porous. This has the effect of making

u p
the bones light enough so that it is easy for the animals to swim

G
back to the surface after a deep dive. The outer shell of the bones

p
was also porous in the ichthyosaur, an extinct prehistoric marine

e
reptile. We can conclude from this that ichthyosaurs were deep

d e
divers. Which one of the following, if true, most weakens the

n
argument?

Sa
A. In most modern and prehistoric marine reptile species that
are not deep divers, the outer shell of the bones is porous.
On average, corporations that encourage frequent social

t a
events in the workplace show higher profits than those that

u p
rarely do. This suggests that the EZ Corporation could boost

G
its profits by having more staff parties during business hours.

p
Which one of the following, if true, most weakens the

e e
argument above?

n d
A. The corporations that encourage frequent social events in

a
the workplace do so to celebrate the high profits that they

S
are already earning.
In the year following an eight-cent increase in the federal tax

t a
on a pack of cigarettes, sales of cigarettes fell ten percent. In

u p
contrast, in the year prior to the tax increase, sales had fallen

G
one percent. The volume of cigarette sales is therefore

p
strongly related to the after-tax price of a pack of cigarettes.

e e
Which of the following, if true, would most strengthen

d
the argument given?

a n
A. The information available to consumers on the health

S
risks of smoking remained largely unchanged in the
period before and after the tax increase.
A survey of clerical workers’ attitudes toward their work identified a

t a
group of secretaries with very positive attitudes. They responded

p
“Strongly agree” to such statements as “I enjoy word processing” and “I

u
like learning new secretarial skills.” These secretaries had been rated

G
by their supervisors as excellent workers—far better than secretaries

e p
whose attitudes were identified as less positive. Clearly these

e
secretaries’ positive attitudes toward their work produced excellent

d
job performance. Which one of the following identifies a reasoning

a n
error in the argument?

S
A. It identifies the secretaries’ positive attitudes as the cause of their
excellent job performance although their attitudes might be an
effect of their performance.
Some people believe that good health is due to luck. However,

t a
studies from many countries indicate a strong correlation

u p
between good health and high educational levels. It is well

G
known that high educational levels allow people to make more

e p
informed lifestyle choices. Thus, research supports the view

e
that good health is largely the result of making informed

n d
lifestyle choices. The reasoning in the argument is most

a
vulnerable to criticism on the grounds that the argument

S
A. overlooks the possibility that the same thing may causally
contribute both to education and to good health.
Premiums for automobile accident insurance are often

t a
higher for red cars than for cars of other colors. To justify

p
these higher charges, insurance companies claim that,

Gu
overall, a greater percentage of red cars are involved in

p
accidents than are cars of any other color. If this claim is

e
true, then lives could undoubtedly be saved by banning red

d e
cars from the roads altogether. The reasoning in the

n
argument is flawed because the argument

a
A. ignores the possibility that drivers who drive recklessly

S
have a preference for red cars
Restaurant manager: In response to requests from our

t a
patrons for vegetarian main dishes, we recently introduced

p
three: an eggplant and zucchini casserole with tomatoes,

Gu
brown rice with mushrooms, and potatoes baked with

p
cheese. The first two are frequently ordered, but no one

e
orders the potato dish, although it costs less than the other

d e
two. Clearly, then, our patrons prefer not to eat potatoes.

n
Which one of the following is an error of reasoning in

a
the restaurant manager's argument?

S
A. treating one of several plausible explanations of a
phenomenon as the only possible explanation
Though sucking zinc lozenges has been promoted as a

t a
treatment for the common cold, research has revealed no

p
consistent effect. Recently, however, a zinc gel applied

u
nasally has been shown to greatly reduce the duration of

G
colds. Since the gel contains zinc in the same form and

e p
concentration as the lozenges, the greater effectiveness of

e
the gel must be due to the fact that cold viruses tend to

n d
concentrate in the nose, not the mouth. Which of the

a
following, if true, most seriously weakens the

S
argument?
A. The zinc gel and the lozenges contain ingredients that
can have an impact on the activity of the zinc
Scientists have determined that an effective way to lower

t a
cholesterol is to eat three servings of whole grains every day.

p
Studies have shown that the cholesterol levels of people who did

u
so were significantly lower after six months than were those of

G
people who did not, even though the cholesterol levels of the two

p
groups were the same before the studies began. Clearly, eating

e
whole grains can have an appreciable effect on cholesterol levels.

d e
The answer to which of the following questions, if true,

n
would be most useful in evaluating the claim about whole

a
grains above?

S
A. Were the two groups of people in the study involved in the
same exercise program?
Community activist: If Morganville wants to keep its central

t a
shopping district healthy, it should prevent the opening of a huge

p
SaveAll discount department store on the outskirts of

u
Morganville. Records from other small towns show that

G
whenever SaveAll has opened a store outside the central

p
shopping district of a small town, within five years the town has

e e
experienced the bankruptcies of more than a quarter of the

d
stores in the shopping district. The answer to which of the

n
following would be most useful for evaluating the

a
community activist’s reasoning?

S
A. In towns with healthy central shopping districts, what
proportion of the stores in those districts suffer bankruptcy
during a typical five-year (without SaveAll) period?
When news periodicals begin forecasting a recession,

t a
people tend to spend less money on nonessential

u p
purchases. Therefore, the perceived threat of a future

G
recession decreases the willingness of people to

p
purchase products that they regard as optional or luxury

e e
goods.

d
Which of the following is an assumption on which the

a n
argument depends?

S
A. Decreased spending on nonessential goods does not
prompt news periodicals to forecast a recession.
Researchers studying the spread of the Black Plague in sixteenth-

t a
century England claim that certain people survived the epidemic

p
because they carried a genetic mutation, known as Delta-32, that

u
is known to prevent the bacteria that causes the Plague from

G
overtaking the immune system. To support this hypothesis, the

p
researchers tested the direct descendants of the residents of an

e
English town where an unusually large proportion of people

d e
survived the Plague. More than half of these descendants tested

n
positive for the mutation Delta-32, a figure nearly three times

a
higher than that found in other locations. The researchers'

S
hypothesis is based on which of the following assumptions?
A. The Plague does not cause genetic mutations such as Delta-32.
Student Advisor: One of our exchange students faced multiple

t a
arguments with her parents over the course of the past year.

p
Not surprisingly, her grade point average (GPA) over the same

u
period showed a steep decline. This is just one example of a

G
general truth: problematic family relationships can cause

p
significant academic difficulties for our students. Which of the

e e
following is required for the Student Advisor to claim that

d
problematic family relationships can cause academic

a n
difficulties?

S
A. The decline in the GPA of the exchange student was not the
reason for the student's arguments with her parents.
CR Basics Part 3
p t a
Gu
e e p
a n d
S
a
Number Percentage Reasoning

u p t
(present in about 10% of CR questions)

p G
d e e
Sa n
a
Number-Percentage concepts:

Misconceptions:

u p t
G
 Percentage increase implies number increase. AND Number increase implies percentage

e p
increase.

e
 Percentage decrease implies number decrease. AND Number decrease implies percentage

n d
decrease.

a
 Large numbers automatically mean large percentages | Large percentages automatically mean

S
large numbers
 Small numbers automatically mean small percentages | Small percentages automatically
mean small numbers.
a
Words that introduce numbers: Amount, Quantity, Sum, Total, Count, Tally

u p t
Words that introduce percentage ideas: Percent, Proportion, Fraction, Ratio, Incidence,

G
Likelihood, Probability, Segment, Share, Rate, Per capita, Per 1000 etc.

e e p
d
Market share is simply the portion of a market that a company controls. Regardless of the size of

n
a market, total market share must always add up to 100%.

Sa
Market share can be measured either in terms of revenues (sales) or units sold.
a
Misconception: % increase automatically means

p t
number increase. Imagine this:

u
Auto manufacturer X increased its United States market share

G
from 10% last year to 25% this year. Therefore, Company X sold

e p
more cars in the United States this year than last.

e
This is true if the size of the U.S. car market stayed the same or

n d
became larger. But if the size of the U.S. car market decreased by

a
enough, the argument would not be valid. Check the table given.

S
It is clear that even though auto manufacturer X’s market share
increased to 25%, because the size of the entire market
decreased significantly, X actually sold fewer cars in the United
States.
a
Misconception: Increasing numbers automatically lead to increasing percentages. Imagine this:

p t
The number of bicycle-related accidents rose dramatically from last month to this month. Therefore,

u
bicycle-related accidents must make up a greater percentage of all road accidents this month.

G
This conclusion can be true, but it does not have to be true, as shown in the table. Thus, even though

e p
the number of bicycle-related accidents tripled, the percentage of total road accidents that were

d e
bicycle-related dropped because the total number of road accidents rose so dramatically.

Sa n
a
Misconception: Large numbers automatically mean large percentages, and small numbers

p t
automatically mean small percentages. Imagine this:

u
In 2003, Porsche sold over 18,000 cars in the United States.

p G
While 18,000 is certainly a large number, it represented only about 1/5 of 1% of total U.S.

e e
car sales in 2003. Remember, the size of a number does not reveal anything about the

d
percentage that number represents unless you know something about the size of the

a n
overall total that number is drawn from.

S
OR … A figure such as 90% sounds impressively large, but if you have 90% of $5, that really
isn’t too impressive, is it?
An automobile dealership’s two car lots have produced remarkably consistent sales figures over the last four

t a
years: in each of those years, the new car lot has contributed 25 percent of dollar sales and 50 percent of

p
profits, and the used car lot has accounted for the balance. Which of the following regarding the past four

Gu
years is most strongly supported by the statements above?

p
A. The used car lot produced lower profits per dollar of sales than the new car lot produced.

d e e
Sa n
a
If we assume the total dollar sales for the dealership was $4,000,000 for the previous year, then the new car lot produced

t
$1,000,000 in sales and the used car lot produced $3,000,000 in sales.

Gu p
p
Now, let’s say that $4,000,000 in total sales yielded a profit of $1,000,000. Since both car lots accounted for 50% of the

e
profits, then both lots produced $500,000 in total profits:

n d e
Sa
As we look at these numbers it becomes immediately clear that the new car lot was significantly more profitable for
every dollar in sales than the used car was. In fact, the used car lot had to sell three times as much as the new lot dollar-
wise to produce the same amount of profit. Answer choice (A): This is the correct answer. As mentioned above, for
every dollar in sales, the used car lot was much less profitable than was the new car lot.
a
Please note:

u p t
G
On each slide, check

e
the bottom

e p
d
notes

Sa n
for explanations,
wherever applicable.
The cost of manufacturing microchips in Country K is 15 percent greater

a
than the cost of manufacturing the same microchips in Country P. Even

t
after customs taxes and delivery fees are considered, it is still cheaper for

p
Country K to import microchips from Country P than to have the

u
microchips manufactured domestically. The claims above, if true, most

G
strongly support which of the following conclusions?

p
A. Customs taxes are less in Country K than they are in Country P.

e
B. It takes 15 percent more time to manufacture a microchip in Country K

e
than it does in Country P.

d
C. The taxes and fees associated with importing microchips from Country

n
P to Country K are less than 15 percent of the cost of manufacturing

a
those microchips in Country K.

S
D. Importing microchips from Country P will reduce employment
opportunities in manufacturing by 15 percent in Country K.
E. Manufacturing costs are the primary consideration for countries when
considering whether to import foreign goods.
a
NOTE … The following questions may have

p t
only one or two or three or four or five

u
G
options ... it is intentional.

e e p
a n d
S
The Mercantile Corporation increased its national market share

t a
last year by 5% compared to its market share two years ago.

p
Which of the following could be true of the overall unit sales of the

Gu
Mercantile Corporation? Select all the possible answers.

p
I. Mercantile Corporation sold fewer units last year than it had

e
sold the prior year.

d e
II. Mercantile Corporation sold the same number of units each of

n
the last two years.

Sa
III.Mercantile Corporation sold more units last year than it had
sold the prior year.
The Mercantile Corporation increased its national market share

t a
last year by 5% compared to its market share two years ago.

p
Which of the following could be true of the overall unit sales of the

Gu
Mercantile Corporation? Select all the possible answers.

p
I. Mercantile Corporation sold fewer units last year than it had

e
sold the prior year. (100000 to 10000)

d e
II. Mercantile Corporation sold the same number of units each of

n
the last two years. (20% of 100000 = 25% of 80000)

Sa
III.Mercantile Corporation sold more units last year than it had
sold the prior year. (Base same)
In today’s mayoral election, West received 1500 votes, compared

t a
with the 1000 votes that he had received in last year’s election.

p
Which of the following could be true of the percentage of the vote

Gu
West won in today’s election compared to the percentage he won
in the last election? Select all the possible answers.

e p
I. West received a greater percentage of the vote today than in

e
the last election.

n d
II. West received a smaller percentage of the vote today than in

a
the last election.

S
III.West received the same percentage of the vote today as in the
last election.
In today’s mayoral election, West received 1500 votes, compared

t a
with the 1000 votes that he had received in last year’s election.

p
Which of the following could be true of the percentage of the vote

Gu
West won in today’s election compared to the percentage he won
in the last election? Select all the possible answers.

e p
I. West received a greater percentage of the vote today than in

e
the last election. (Same base)

n d
II. West received a smaller percentage of the vote today than in

a
the last election. (1500/15000 vs. 1000/2000)

S
III.West received the same percentage of the vote today as in the
last election. (1500/15000 vs. 1000/10000)
Halstead’s and McGrady’s are competing furniture stores, each of which

t a
carries exactly one type of couch. Next week Halstead’s will have its annual

p
holiday sale, during which every piece of furniture in the store is to be marked

u
down by 60%. McGrady’s has just announced a competing sale, in which

G
various products will be marked down by 30%. Which of the following could be

p
true of Halstead’s couch price compared with McGrady’s during next week’s sale?

e
Select all the possible answers.

d e
I. A couch purchased at Halstead’s will cost less than a couch at McGrady’s.

n
II. A couch purchased at Halstead’s will cost more than a couch at McGrady’s.

Sa
III.A couch purchased at Halstead’s will cost the same as a couch at McGrady’s.
Halstead’s and McGrady’s are competing furniture stores, each of which

t a
carries exactly one type of couch. Next week Halstead’s will have its annual

p
holiday sale, during which every piece of furniture in the store is to be marked

u
down by 60%. McGrady’s has just announced a competing sale, in which

G
various products will be marked down by 30%. Which of the following could be

p
true of Halstead’s couch price compared with McGrady’s during next week’s sale?

e
Select all the possible answers.

d e
I. A couch purchased at Halstead’s will cost less than a couch at McGrady’s.

n
(initial price 100 for both)

Sa
II. A couch purchased at Halstead’s will cost more than a couch at McGrady’s.
(200 down 60% is 80 | 100 down 30% is 70)
III.A couch purchased at Halstead’s will cost the same as a couch at McGrady’s.
0.4x = 0.7y or 4x = 7y, so, if x = 70 and y = 40 this can work out
In response to brisk sales, a certain car dealership increased the price of the

t a
Cheetah, its best-selling sport utility vehicle, by 25% on January 1. In

p
February, after no other price changes had been implemented, the

u
dealership held a special sale during which the price of every car was

G
marked down by 20%. Which of the following could be true of the price of the

p
Cheetah during the February sale compared with the price on December 31

e
(just prior to the January 1 price change)? Select all the possible answers.

d e
I. The price of the Cheetah was higher during the sale than it had been on

n
December 31.

a
II. The price of the Cheetah was lower during the sale than it had been on

S
December 31.
III. The price of the Cheetah was the same during the sale as it had been on
December 31.
In response to brisk sales, a certain car dealership increased the price of the

t a
Cheetah, its best-selling sport utility vehicle, by 25% on January 1. In

p
February, after no other price changes had been implemented, the

u
dealership held a special sale during which the price of every car was

G
marked down by 20%. Which of the following could be true of the price of the

p
Cheetah during the February sale compared with the price on December 31

e
(just prior to the January 1 price change)? Select all the possible answers.

d e
I. The price of the Cheetah was higher during the sale than it had been on

n
December 31. Impossible

a
II. The price of the Cheetah was lower during the sale than it had been on

S
December 31. Impossible
III. The price of the Cheetah was the same during the sale as it had been on
December 31. ONLY this is correct
a
Last year, Davis, Acme Company’s top salesperson, was responsible for 25% of Acme’s

t
total sales. This year Davis is credited with 35% of Acme’s total sales, which have

u p
decreased overall compared to last year’s total sales. Which of the following could be true

G
of Davis’ sales this year as compared with Davis’ sales from last year? Select all the

p
possible answers.

e e
I. Davis’ total sales were greater this year.

d
II. Davis’ total sales were greater last year.

a n
III. Davis’ total sales were the same over the last two years.

S
a
Last year, Davis, Acme Company’s top salesperson, was responsible for 25% of Acme’s

t
total sales. This year Davis is credited with 35% of Acme’s total sales, which have

u p
decreased overall compared to last year’s total sales. Which of the following could be true

G
of Davis’ sales this year as compared with Davis’ sales from last year? Select all the

p
possible answers.

e e
I. Davis’ total sales were greater this year. (25% of 10000 versus 35% of 8000)

d
II. Davis’ total sales were greater last year. (25% of 10000 versus 35% of 1000)

a n
III. Davis’ total sales were the same over the last two years. (35% of 100 = 25% of 140)

S
Corporate Officer: Last year was an unusually poor one for our chemical division, which has

a
traditionally contributed about 60 percent of the corporation’s profits. It is therefore

t
encouraging that the pharmaceutical division is growing stronger: it contributed 45 percent

p
of the corporation’s profits, up from 20 percent the previous year.

u
On the basis of the facts stated, which of the following is the best counter to the claim

G
that the pharmaceutical division is growing stronger?
A. The increase in the pharmaceutical division’s contribution to corporation profits could

p
have resulted largely from the introduction of single, important new product.

e
B. In multidivisional corporations that have pharmaceutical divisions, over half of the

e
corporation’s profits usually come from the pharmaceuticals.

d
C. The percentage of the corporation’s profits attributable to the pharmaceutical division

n
could have increased even if that division’s performance had not improved.

a
D. The information cited does not make it possible to determine whether the 20 percent

S
share of profits cited was itself an improvement over the year before.
E. The information cited does not make it possible to compare the performance of the
chemical and pharmaceutical divisions in of the percent of total profits attributable to
each.
Corporate Officer: Last year was an unusually poor one for our chemical division, which has

a
traditionally contributed about 60 percent of the corporation’s profits. It is therefore

t
encouraging that the pharmaceutical division is growing stronger: it contributed 45 percent

p
of the corporation’s profits, up from 20 percent the previous year.

u
On the basis of the facts stated, which of the following is the best counter to the claim

G
that the pharmaceutical division is growing stronger?
A. The increase in the pharmaceutical division’s contribution to corporation profits could

p
have resulted largely from the introduction of single, important new product.

e
B. In multidivisional corporations that have pharmaceutical divisions, over half of the

e
corporation’s profits usually come from the pharmaceuticals.

d
C. The percentage of the corporation’s profits attributable to the pharmaceutical division

n
could have increased even if that division’s performance had not improved. Earlier: 1000

a
 20% = 200 (pharma) | 100  45% = 45 (pharma)

S
D. The information cited does not make it possible to determine whether the 20 percent
share of profits cited was itself an improvement over the year before.
E. The information cited does not make it possible to compare the performance of the
chemical and pharmaceutical divisions in of the percent of total profits attributable to
each.
Studies show that impoverished families give away a larger percentage

t a
of their income in charitable donations than do wealthy families. As a

p
result, fundraising consultants recommend that charities direct their

u
marketing efforts toward individuals and families from lower

G
socioeconomic classes in order to maximize the dollar value of

e p
incoming donations. Which of the following best explains why the

e
consultants' reasoning is flawed?

n d
A. Percentage of income is not necessarily indicative of absolute

a
dollar value.

S
Students from outside the province of Markland, who in any given

t a
academic year pay twice as much tuition fee each as do students from

p
Markland, had traditionally accounted for at least two-thirds of the

u
enrollment at Central Markland College. Over the past 10 years academic

G
standards at the college have risen, and the proportion of students who
are not Marklanders has dropped to around 40 percent. Which one of the

e p
following can be properly inferred from the statements above?

e
A. Over the past 10 years, the number of students from Markland

d
increased and the number of students from outside Markland

n
decreased.

a
B. If the college’s per capita revenue from tuition has remained the same,

S
tuition fees have increased over the past 10 years.
The number of North American children who are

t a
obese—that is, who have more body fat than do 85

u p
percent of North American children their age—is

G
steadily increasing, according to four major studies

e p
conducted over the past 15 years. If the finding

e
reported above is correct, it can be properly

n d
concluded that:

Sa
A. the number of North American children who are
not obese increased over the past 15 years
For a ten-month period, the total monthly sales of new cars within the country

t a
of Calistan remained constant. During this period the monthly sales of new cars

p
manufactured by Marvel Automobile Company doubled, and its share of the new

u
car market within Calistan increased correspondingly. At the end of this period,

G
emission standards were imposed on new cars sold within Calistan. During the

p
three months following this imposition, Marvel Automobile Company’s share of

e
the Calistan market declined substantially even though its monthly sales within

e
Calistan remained constant at the level reached in the last month of the ten-

n d
month period. If the statements above are true, which one of the following

a
CANNOT be true?

S
A. The total monthly sales within Calistan of new cars by companies other than
Marvel Automobile Company decreased over the three months following the
imposition of the emission standards.
For a ten-month period, the total monthly sales of new cars within the country

t a
of Calistan remained constant. During this period the monthly sales of new cars

p
manufactured by Marvel Automobile Company doubled, and its share of the new

u
car market within Calistan increased correspondingly. At the end of this period,

G
emission standards were imposed on new cars sold within Calistan. During the

p
three months following this imposition, Marvel Automobile Company’s share of

e
the Calistan market declined substantially even though its monthly sales within

e
Calistan remained constant at the level reached in the last month of the ten-

n d
month period. If the statements above are true, which one of the following

a
CANNOT be true?

S
A. The total monthly sales within Calistan of new cars by companies other than
Marvel Automobile Company decreased over the three months following the
imposition of the emission standards.
In Eastland, from 2000 to 2005, the total consumption of

t a
fish increased by 4.5 percent, and the total consumption of

p
poultry products increased by 9.0 percent. During this

u
time, the population of Eastland increased by 6 percent, in

G
part due to new arrivals from surrounding areas. Which of

p
the following can one infer based on the statements

e e
above?

d
A. The per capita consumption of poultry in Eastland was

a n
higher in 2005 than it was in 2000.

S
Box office receipts for independent movies for the first half

t a
of this year have increased by 20 percent over the total

p
receipts for independent movies for all of last year. Last

u
year, 50 independent movies were released, while so far

G
this year only 20 independent movies have been released.

p
The number of independent movies slated for release in

e e
the second half of this year is roughly equal to the number

d
released so far. If the statements above are true, which

a n
of the following must be true?

S
A. The average revenues of the independent films released
during the first half of this year is greater than that of all
independent films released last year.
From 1973 to 1989 total energy use in this country increased

t a
less than 10 percent. However, the use of electrical energy in this

p
country during this same period grew by more than 50 percent,

u
as did the gross national product—the total value of all goods

G
and services produced in the nation. If the statements above

p
are true, then which one of the following must also be true?

e
A. Most of the energy used in this country in 1989 was

e
electrical energy.

n d
B. From 1973 to 1989 there was a decline in the use of energy

a
other than electrical energy in this country.

S
C. From 1973 to 1989 there was an increase in the proportion
of energy use in this country that consisted of electrical
energy use.
From 1973 to 1989 total energy use in this country increased

t a
less than 10 percent. However, the use of electrical energy in this

p
country during this same period grew by more than 50 percent,

u
as did the gross national product—the total value of all goods

G
and services produced in the nation. If the statements above

p
are true, then which one of the following must also be true?

e
A. Most of the energy used in this country in 1989 was

e
electrical energy.

n d
B. From 1973 to 1989 there was a decline in the use of energy

a
other than electrical energy in this country.

S
C. From 1973 to 1989 there was an increase in the
proportion of energy use in this country that consisted
of electrical energy use.
Ditrama is a federation made up of three autonomous regions: Korva, Mitro,

a
and Guadar. Under the federal revenue-sharing plan, each region receives a

p t
share of federal revenues equal to the share of the total population of

u
Ditrama residing in that region, as shown by a yearly population survey.

G
Last year, the percentage of federal revenues Korva received for its share
decreased somewhat even though the population survey on which the

p
revenue-sharing was based showed that Korva’s population had increased.

e e
If the statements above are true, which one of the following must also

d
have been shown by the population survey on which last year’s

n
revenue-sharing in Ditrama was based?

a
A. The populations of Mitro and Guadar each increased by a percentage

S
that exceeded the percentage by which the population of Korva
increased.
B. Korva’s population grew by a smaller percentage than did the
population of at least one of the other two autonomous regions.
Ditrama is a federation made up of three autonomous regions: Korva, Mitro,

a
and Guadar. Under the federal revenue-sharing plan, each region receives a

p t
share of federal revenues equal to the share of the total population of

u
Ditrama residing in that region, as shown by a yearly population survey.

G
Last year, the percentage of federal revenues Korva received for its share
decreased somewhat even though the population survey on which the

p
revenue-sharing was based showed that Korva’s population had increased.

e e
If the statements above are true, which one of the following must also

d
have been shown by the population survey on which last year’s

n
revenue-sharing in Ditrama was based?

a
A. The populations of Mitro and Guadar each increased by a percentage

S
that exceeded the percentage by which the population of Korva
increased.
B. Korva’s population grew by a smaller percentage than did the
population of at least one of the other two autonomous regions.
The restaurant business wastes more energy than any other

t a
industry in the United States. Nearly 80 percent of the $10 billion

p
spent on energy by the restaurant industry each year is

u
squandered by the use of inefficient equipment. At the same

G
time, approximately 70 percent of restaurants in the United

p
States are small businesses that are usually too cash poor to

e
invest in energy-efficient technology. Which of the following

e
statements draws the most reliable conclusion from the

n d
information above?

a
A. The replacement of inefficient equipment represents the

S
largest potential source of energy savings for the restaurant
industry.
There is relatively little room for growth in the overall carpet

t a
market, which is tied to the size of the population. Most who

p
purchase carpet do so only once or twice, first in their twenties or

u
thirties, and then perhaps again in their fifties or sixties. Thus as

G
the population ages, companies producing carpet will be able to

p
gain market share in the carpet market only through purchasing

e
competitors, and not through more aggressive marketing.

d e
Weaken

n
A. Two of the three mergers in the industry’s last ten years led to

a
a decline in profits and revenues for the newly merged

S
companies.
B. Price reductions, achieved by cost-cutting in production, by
some of the dominant firms in the carpet market are causing
other producers to leave the market altogether.
In the United States, of the people who moved from one state to

t a
another when they retired, the percentage who retired to Florida has

p
decreased by three percentage points over the past ten years. Since

u
many local businesses in Florida cater to retirees, these declines are

G
likely to have a noticeably negative economic effect on these

p
businesses and therefore on the economy of Florida. Weaken

e
A. The number of people who moved from one state to another

d e
when they retired has increased significantly over the past ten

n
years.

a
B. The number of people who left Florida when they retired to live

S
in another state was greater last year than it was ten years ago.
C. Florida attracts more people who move from one state to another
when they retire than does any other state.
In the past, most children who went sledding in the

t a
winter snow in Verland used wooden sleds with runners

p
and steering bars. Ten years ago, smooth plastic sleds

u
became popular; they go faster than wooden sleds but are

G
harder to steer and slow. The concern that plastic sleds

e p
are more dangerous is clearly borne out by the fact that

e
the number of children injured while sledding was much

d
higher last winter than it was ten years ago. Which of the

a n
following, if true in Verland, most seriously

S
undermines the force of the evidence cited?
A. Plastic sleds can be used in a much wider variety of
snow conditions than wooden sleds can.
The violent crime rate (number of violent crimes per 1,000

t a
residents) in Meadowbrook is 60 percent higher now than it was

p
four years ago. The corresponding increase for Parkdale is only

u
10 percent. These figures support the conclusion that residents

G
of Meadowbrook are more likely to become victims of violent

p
crime than are residents of Parkdale. The argument above is

e e
flawed because it fails to take into account

d
A. how the rate of population growth in Meadowbrook over the

a n
past four years compares to the corresponding rate for

S
Parkdale
B. the violent crime rates in Meadowbrook and Parkdale four
years ago
Profits for one of Company X’s flagship products have been declining slowly for

a
several years. The CFO investigated and determined that inflation has raised

t
the cost of producing the product but consumers who were surveyed reported

p
that they weren’t willing to pay more than the current price. As a result, the

u
CFO recommended that the company stop producing this product because the
CEO only wants products whose profit margins are increasing.

G
The answer to which of the following questions would be most useful in

p
evaluating whether the CFO’s decision to divest the company of its

e
flagship product is warranted?

e
A. Does the company have new and profitable products available with which to
replace the flagship product?

d
B. What percentage of Company X’s revenues is represented by sales of the

n
flagship product in question?

a
C. Are there additional features which could be added to the product and for

S
which consumers might be willing to pay a higher price?
D. Will the rest of Company X’s management team agree with the CFO’s
recommendation?
E. Is there a way to reduce the cost to produce the flagship product?
If the appropriate timely surgery is not performed on someone who has suffered

a
from a heart attack, the outcome can be potentially fatal; consequently, patients

t
with symptoms strongly suggesting a heart attack are almost always made to

p
undergo a surgery, whether they have actually had a heart attack or not. The

u
appropriate surgery is extremely low-risk but is performed unnecessarily in
about 20 percent of all cases. A newly developed internal scan to determine

G
whether someone with symptoms of heart attack has actually had a heart attack

p
produces absolutely correct diagnosis in 98% of the cases. Clearly, using this

e
scan, doctors can largely avoid unnecessary surgeries related to heart attacks,

e
without, however, performing any fewer necessary surgeries.
Which of the following assumption is required for the conclusion to be

d
always true?

n
A. the misdiagnoses produced by this scan are always instances of attributing

a
heart attack to someone who has not had it

S
B. the misdiagnoses produced by this scan are never instances of attributing
heart attack to someone who has not had it
C. all of the patients who are diagnosed with this scan as having had a heart
attack have actually had a heart attack
a
Miscellaneous CR questions involving
Math:
u p t
p G
e e
numbers / rates / ratios / per-capita income / employment etc.

a n d
S
For several years, per capita expenditure on prescription drugs in

t a
Voronia rose by fifteen percent or more annually. In order to curb

u p
these dramatic increases, the ministry of health prohibited drug

G
manufacturers from raising any of their existing products' prices.

p
Even though use of prescription drugs did not expand after this

e
price freeze, per capita expenditure for prescription drugs

d e
continued to increase by a substantial percentage each year.

n
Which of the following, if true, most helps to explain why the

Sa
ministry's action did not achieve its goal?
A. The population of Voronia rose steadily throughout the period.
Finding of a survey of Systems magazine subscribers: Thirty

t a
percent of all merchandise orders placed by subscribers in

p
response to advertisements in the magazine last year were

u
placed by subscribers under age thirty-five.

G
Finding of a survey of advertisers in Systems magazine:

p
Most of the merchandise orders placed in response to

e
advertisements in Systems last year were placed by people

d e
under age thirty-five.

n
For both of the findings to be accurate, which of the

a
following must be true?

S
A. Last year many people who placed orders for merchandise
in response to advertisements in Systems were not
subscribers to the magazine.
Guidebook writer: I have visited hotels throughout the country

t a
and have noticed that in those built before 1930 the quality of the

p
original carpentry work is generally superior to that in hotels built

u
afterward. Clearly carpenters working on hotels before 1930

G
typically worked with more skill, care, and effort than carpenters

e p
who have worked on hotels built subsequently. Which of the

e
following, if true, most seriously weakens the guidebook

d
writer's argument?

a n
A. The better the quality of original carpentry in a building, the

S
less likely that building is to fall into disuse and be demolished.
Because postage rates are rising, Home Decorator magazine plans to maximize

a
its profits by reducing by one half the number of issues it publishes each year.

t
The quality of articles, the number of articles published per year, and the

p
subscription price will not change. Market research shows that neither

u
subscribers nor advertisers will be lost if the magazine's plan is instituted.

G
Which of the following, if true, provides the strongest evidence that the
magazine's profits are likely to decline if the plan is instituted?

p
A. With the new postage rates, a typical issue under the proposed plan would

e
cost about one-third more to mail than a typical current issue would.

e
B. The majority of the magazine's subscribers are less concerned about a

d
possible reduction in the quantity of the magazine's articles than about a

n
possible loss of the current high quality of its articles.

a
C. Many of the magazine's long-time subscribers would continue their

S
subscriptions even if the subscription price were increased.
D. Most of the advertisers that purchase advertising space in the magazine will
continue to spend the same amount on advertising per issue as they have in
the past.
E. Production costs for the magazine are expected to remain stable.
Coal output per miner in Tribnia is twice what it was five years ago, even

a
though no new mines have opened.

t
Which of the following can be properly concluded from the statement

p
about coal output per miner in the passage?

u
A. If the number of miners working in Tribnian coal mines has remained

G
constant in the past five years, Tribnia’s total coal production has doubled in
that period of time.

p
B. Any individual Tribnian coal mine that achieved an increase in overall

e
output in the past five years has also experienced an increase in output per

e
miner.

d
C. If any new coal mines had opened in Tribnia in the past five years, then the

n
increase in output per miner would have been even greater than it actually

a
was.

S
D. If any individual Tribnian coal mine has not increased its output per miner
in the past five years, then that mine’s overall output has declined or
remained constant.
E. In Tribnia the cost of producing a given quantity of coal has declined over
the past five years.
In a poll of a representative sample of a province’s residents, the provincial capital

a
was the city most often selected as the best place to live in that province. Since the

t
capital is also the largest of that province’s many cities, the poll shows that most

p
residents of that province generally prefer life in large cities to life in small cities. The

u
argument is most vulnerable to the criticism that it

G
A. overlooks the possibility that what is true of the residents of the province may

p
not be true of other people

e
B. does not indicate whether most residents of other provinces also prefer life in

e
large cities to life in small cities

d
C. takes for granted that when people are polled for their preferences among cities,

n
they tend to vote for the city that they think is neither the best nor the worst
place to live

a
D. overlooks the possibility that the people who preferred small cities over the

S
provincial capital did so not because of their general feelings about the sizes of
cities, but because of their general feelings about capital cities
E. overlooks the possibility that most people may have voted for small cities even
though a large city received more votes than any other single city
A controversial program rewards prison inmates who behave particularly well in

a
prison by giving them the chance to receive free cosmetic plastic surgery performed by

t
medical students. The program is obviously morally questionable, both in its

p
assumptions about what inmates might want and in its use of the prison population to

u
train future surgeons. Putting these moral issues aside, however, the surgery clearly has
a powerful rehabilitative effect, as is shown by the fact that, among recipients of the

G
surgery, the proportion who are convicted of new crimes committed after release is
only half that for the prison population as a whole. A flaw in the reasoning of the

p
passage is that it

e
A. allows moral issues to be a consideration in presenting evidence about matters of

e
fact

d
B. dismisses moral considerations on the grounds that only matters of fact are

n
relevant

a
C. labels the program as "controversial" instead of discussing the issues that give rise

S
to controversy
D. asserts that the rehabilitation of criminals is not a moral issue
E. relies on evidence drawn from a sample that there is reason to believe is
unrepresentative
a
Assumption Centrality Technique

u p t
p G
d e e
Sa n
a
Imagine the argument:

u p t
p G
As Ruparia’s football team has been performing

e e
very badly for the last three World Cups, it

d
n
should not be allowed to play in the next few

Sa World Cups.
Ruparia’s football team has been performing very badly for the last three World

t a
Cups, THEREFORE it should not be allowed to play for the next few World Cups.

u
What are all possible assumptions in this argument?

G p
e e p
a n d
S
Ruparia’s football team has been performing very badly for the last three World

t a
Cups, THEREFORE it should not be allowed to play for the next few World Cups.

u
What are all possible weakeners in this argument?

G p
e e p
a n d
S
Ruparia’s football team has been performing very badly for the last three World

t a
Cups, THEREFORE it should not be allowed to play for the next few World Cups.

u
What are all possible strengtheners in this argument?

G p
e e p
a n d
S
Ruparia’s football team has been performing very badly for the last three World

t a
Cups, THEREFORE it should not be allowed to play for the next few World Cups.

u
What are all possible flaws in this argument?

G p
e e p
a n d
S
Ruparia’s football team has been performing very badly for the last three World

t a
Cups, THEREFORE it should not be allowed to play for the next few World Cups.

u
The answer to which question(s) can help us evaluate
the validity of this conclusion better?
G p
e e p
a n d
S
Further study after CR Basics 2 and 3

CR Class Sessions Guide PDFp t a


G u (full)

e e p
a n d
Conditional Reasoning (Full)

You might also like